Question and Answers Forum

All Questions      Topic List

Number Theory Questions

Previous in All Question      Next in All Question      

Previous in Number Theory      Next in Number Theory      

Question Number 9025 by Rasheed Soomro last updated on 15/Nov/16

Determine number/s that is/are comprised  of four distinct prime factors such that  difference of largest and smallest prime  factors is equal to the sum of remaining  two factors.                  _(Propsed by Rasheed Soomro)

$$\mathrm{Determine}\:\mathrm{number}/\mathrm{s}\:\mathrm{that}\:\mathrm{is}/\mathrm{are}\:\mathrm{comprised} \\ $$$$\mathrm{of}\:\mathrm{four}\:\mathrm{distinct}\:\mathrm{prime}\:\mathrm{factors}\:\mathrm{such}\:\mathrm{that} \\ $$$$\mathrm{difference}\:\mathrm{of}\:\mathrm{largest}\:\mathrm{and}\:\mathrm{smallest}\:\mathrm{prime} \\ $$$$\mathrm{factors}\:\mathrm{is}\:\mathrm{equal}\:\mathrm{to}\:\mathrm{the}\:\mathrm{sum}\:\mathrm{of}\:\mathrm{remaining} \\ $$$$\mathrm{two}\:\mathrm{factors}.\:\:\:\:\:\:\:\:\:\:\:\:\:\:\:\:\:\:_{\mathrm{Propsed}\:\mathrm{by}\:\mathrm{Rasheed}\:\mathrm{Soomro}} \\ $$

Commented by FilupSmith last updated on 15/Nov/16

Do you mean:  n=p_1 p_2 p_3 p_4   p_a >p_(a+1)   p_1 −p_4 =p_2 +p_3   p_a ∈P, p_a ≠p_(a+i) ∀i/{0}  n=?

$$\mathrm{Do}\:\mathrm{you}\:\mathrm{mean}: \\ $$$${n}={p}_{\mathrm{1}} {p}_{\mathrm{2}} {p}_{\mathrm{3}} {p}_{\mathrm{4}} \\ $$$${p}_{{a}} >{p}_{{a}+\mathrm{1}} \\ $$$${p}_{\mathrm{1}} −{p}_{\mathrm{4}} ={p}_{\mathrm{2}} +{p}_{\mathrm{3}} \\ $$$${p}_{{a}} \in\mathbb{P},\:{p}_{{a}} \neq{p}_{{a}+{i}} \forall{i}/\left\{\mathrm{0}\right\} \\ $$$${n}=? \\ $$

Commented by Rasheed Soomro last updated on 16/Nov/16

Yes Filup I mean that.

$$\mathrm{Yes}\:\mathrm{Filup}\:\mathrm{I}\:\mathrm{mean}\:\mathrm{that}. \\ $$

Commented by FilupSmith last updated on 16/Nov/16

working/playing around  x=p_1 p_2 p_3 p_4      p_1 −p_4 =p_2 +p_3   p_1 =p_2 +p_3 +p_4   ∴ x=(p_2 +p_3 +p_4 )p_2 p_3 p_4   ∴p_2 +p_3 +p_4 ∈P    p_2 +p_3 +p_4 <p_2 <p_3 <p_4   p_2 <p_2 −p_3 <−p_4 <−p_3      p_2  is +ve  p_2 −p_3  is −ve  −p_4   and  −p_3    are −ve     ∴i dont think such number exists  as the equality is not possible  working/playing around

$${working}/{playing}\:{around} \\ $$$${x}={p}_{\mathrm{1}} {p}_{\mathrm{2}} {p}_{\mathrm{3}} {p}_{\mathrm{4}} \\ $$$$\: \\ $$$${p}_{\mathrm{1}} −{p}_{\mathrm{4}} ={p}_{\mathrm{2}} +{p}_{\mathrm{3}} \\ $$$${p}_{\mathrm{1}} ={p}_{\mathrm{2}} +{p}_{\mathrm{3}} +{p}_{\mathrm{4}} \\ $$$$\therefore\:{x}=\left({p}_{\mathrm{2}} +{p}_{\mathrm{3}} +{p}_{\mathrm{4}} \right){p}_{\mathrm{2}} {p}_{\mathrm{3}} {p}_{\mathrm{4}} \\ $$$$\therefore{p}_{\mathrm{2}} +{p}_{\mathrm{3}} +{p}_{\mathrm{4}} \in\mathbb{P} \\ $$$$ \\ $$$${p}_{\mathrm{2}} +{p}_{\mathrm{3}} +{p}_{\mathrm{4}} <{p}_{\mathrm{2}} <{p}_{\mathrm{3}} <{p}_{\mathrm{4}} \\ $$$${p}_{\mathrm{2}} <{p}_{\mathrm{2}} −{p}_{\mathrm{3}} <−{p}_{\mathrm{4}} <−{p}_{\mathrm{3}} \\ $$$$\: \\ $$$${p}_{\mathrm{2}} \:\mathrm{is}\:+\mathrm{ve} \\ $$$${p}_{\mathrm{2}} −{p}_{\mathrm{3}} \:\mathrm{is}\:−\mathrm{ve} \\ $$$$−{p}_{\mathrm{4}} \:\:\mathrm{and}\:\:−{p}_{\mathrm{3}} \:\:\:\mathrm{are}\:−\mathrm{ve} \\ $$$$\: \\ $$$$\therefore{i}\:{dont}\:{think}\:{such}\:{number}\:{exists} \\ $$$${as}\:{the}\:{equality}\:{is}\:{not}\:{possible} \\ $$$${working}/{playing}\:{around} \\ $$

Commented by mrW last updated on 16/Nov/16

when I take a look at the table of   prime numbers I think there  should be infinitely many such  numbers, such as  x=3×5×11×19  x=5×7×11×23  x=3×7×13×23  etc.

$${when}\:{I}\:{take}\:{a}\:{look}\:{at}\:{the}\:{table}\:{of}\: \\ $$$${prime}\:{numbers}\:{I}\:{think}\:{there} \\ $$$${should}\:{be}\:{infinitely}\:{many}\:{such} \\ $$$${numbers},\:{such}\:{as} \\ $$$${x}=\mathrm{3}×\mathrm{5}×\mathrm{11}×\mathrm{19} \\ $$$${x}=\mathrm{5}×\mathrm{7}×\mathrm{11}×\mathrm{23} \\ $$$${x}=\mathrm{3}×\mathrm{7}×\mathrm{13}×\mathrm{23} \\ $$$${etc}. \\ $$

Commented by Rasheed Soomro last updated on 16/Nov/16

^• THαnks of you both!  ^• Good attack of Filup  to the problem but  failed in the end!  ^• I also think that such numbers are infinite  but  Can we prove that such numbers are infimite?

$$\:^{\bullet} \mathcal{TH}\alpha{nks}\:{of}\:{you}\:{both}! \\ $$$$\:^{\bullet} \mathrm{Good}\:\mathrm{attack}\:\mathrm{of}\:\mathrm{Filup}\:\:\mathrm{to}\:\mathrm{the}\:\mathrm{problem}\:\mathrm{but} \\ $$$$\mathrm{failed}\:\mathrm{in}\:\mathrm{the}\:\mathrm{end}! \\ $$$$\:^{\bullet} \mathrm{I}\:\mathrm{also}\:\mathrm{think}\:\mathrm{that}\:\mathrm{such}\:\mathrm{numbers}\:\mathrm{are}\:\mathrm{infinite} \\ $$$$\mathrm{but}\:\:\boldsymbol{\mathrm{Can}}\:\boldsymbol{\mathrm{we}}\:\boldsymbol{\mathrm{prove}}\:\boldsymbol{\mathrm{that}}\:\boldsymbol{\mathrm{such}}\:\boldsymbol{\mathrm{numbers}}\:\boldsymbol{\mathrm{are}}\:\boldsymbol{\mathrm{infimite}}? \\ $$$$ \\ $$

Commented by FilupSmith last updated on 16/Nov/16

I wonder where I went wrong

$$\mathrm{I}\:\mathrm{wonder}\:\mathrm{where}\:\mathrm{I}\:\mathrm{went}\:\mathrm{wrong} \\ $$

Commented by FilupSmith last updated on 16/Nov/16

I think my mistake was that in my math,  p_1  is actually >p_2 , but i missthought  it as p_1 <p_2 , or vica versa.     Everything up to  x=(p_2 +p_3 +p_4 )p_2 p_3 p_4   is correct. BUT,  it should be that:  p_1 >p_2 >p_3 >p_3 .

$$\mathrm{I}\:\mathrm{think}\:\mathrm{my}\:\mathrm{mistake}\:\mathrm{was}\:\mathrm{that}\:\mathrm{in}\:\mathrm{my}\:{m}\mathrm{ath}, \\ $$$${p}_{\mathrm{1}} \:\mathrm{is}\:\mathrm{actually}\:>{p}_{\mathrm{2}} ,\:\mathrm{but}\:\mathrm{i}\:\mathrm{missthought} \\ $$$$\mathrm{it}\:\mathrm{as}\:{p}_{\mathrm{1}} <{p}_{\mathrm{2}} ,\:\mathrm{or}\:\mathrm{vica}\:\mathrm{versa}. \\ $$$$\: \\ $$$$\mathrm{Everything}\:\mathrm{up}\:\mathrm{to} \\ $$$${x}=\left({p}_{\mathrm{2}} +{p}_{\mathrm{3}} +{p}_{\mathrm{4}} \right){p}_{\mathrm{2}} {p}_{\mathrm{3}} {p}_{\mathrm{4}} \\ $$$$\mathrm{is}\:\mathrm{correct}.\:\mathrm{BUT}, \\ $$$$\mathrm{it}\:\boldsymbol{\mathrm{should}}\:\mathrm{be}\:\mathrm{that}: \\ $$$${p}_{\mathrm{1}} >{p}_{\mathrm{2}} >{p}_{\mathrm{3}} >{p}_{\mathrm{3}} . \\ $$

Terms of Service

Privacy Policy

Contact: info@tinkutara.com